For a sequence of positive real numbers converging to a limit (which is not equal to 0), show that infimum...












0












$begingroup$


I think to do this I either need to prove the infimum is a minumum or a limit but I'm not sure how.



I have tried this:



By definition of the infimum:
" Let A be a subset of the real numbers and b be a real number.
Then b is the infimum of A if:
(i) b≤a for all a ∈ A
(ii) c≤a for all a ∈ A will imply that c≤b."



We know for all a ∈ A, a>0.
If we consider the second case, this means the infimum (b) is such that 0≤b.



We can assume for a contradiction that b = 0 in order to leave just the case 0 < b.
b cannot be the minimum of the set, as all a > 0.
because all the terms are positive, for the infimum to be 0, it means the limit of the series is zero (and the sequence must be decreasing), but L cannot be equal to 0 so we have a contradiction.



This highlighted bit is the jump I'm struggling with.










share|cite|improve this question











$endgroup$












  • $begingroup$
    What have you tried so far ?
    $endgroup$
    – Digitalis
    Dec 3 '18 at 21:24
















0












$begingroup$


I think to do this I either need to prove the infimum is a minumum or a limit but I'm not sure how.



I have tried this:



By definition of the infimum:
" Let A be a subset of the real numbers and b be a real number.
Then b is the infimum of A if:
(i) b≤a for all a ∈ A
(ii) c≤a for all a ∈ A will imply that c≤b."



We know for all a ∈ A, a>0.
If we consider the second case, this means the infimum (b) is such that 0≤b.



We can assume for a contradiction that b = 0 in order to leave just the case 0 < b.
b cannot be the minimum of the set, as all a > 0.
because all the terms are positive, for the infimum to be 0, it means the limit of the series is zero (and the sequence must be decreasing), but L cannot be equal to 0 so we have a contradiction.



This highlighted bit is the jump I'm struggling with.










share|cite|improve this question











$endgroup$












  • $begingroup$
    What have you tried so far ?
    $endgroup$
    – Digitalis
    Dec 3 '18 at 21:24














0












0








0


0



$begingroup$


I think to do this I either need to prove the infimum is a minumum or a limit but I'm not sure how.



I have tried this:



By definition of the infimum:
" Let A be a subset of the real numbers and b be a real number.
Then b is the infimum of A if:
(i) b≤a for all a ∈ A
(ii) c≤a for all a ∈ A will imply that c≤b."



We know for all a ∈ A, a>0.
If we consider the second case, this means the infimum (b) is such that 0≤b.



We can assume for a contradiction that b = 0 in order to leave just the case 0 < b.
b cannot be the minimum of the set, as all a > 0.
because all the terms are positive, for the infimum to be 0, it means the limit of the series is zero (and the sequence must be decreasing), but L cannot be equal to 0 so we have a contradiction.



This highlighted bit is the jump I'm struggling with.










share|cite|improve this question











$endgroup$




I think to do this I either need to prove the infimum is a minumum or a limit but I'm not sure how.



I have tried this:



By definition of the infimum:
" Let A be a subset of the real numbers and b be a real number.
Then b is the infimum of A if:
(i) b≤a for all a ∈ A
(ii) c≤a for all a ∈ A will imply that c≤b."



We know for all a ∈ A, a>0.
If we consider the second case, this means the infimum (b) is such that 0≤b.



We can assume for a contradiction that b = 0 in order to leave just the case 0 < b.
b cannot be the minimum of the set, as all a > 0.
because all the terms are positive, for the infimum to be 0, it means the limit of the series is zero (and the sequence must be decreasing), but L cannot be equal to 0 so we have a contradiction.



This highlighted bit is the jump I'm struggling with.







real-analysis limits supremum-and-infimum






share|cite|improve this question















share|cite|improve this question













share|cite|improve this question




share|cite|improve this question








edited Dec 4 '18 at 10:30







Laura

















asked Dec 3 '18 at 21:19









LauraLaura

112




112












  • $begingroup$
    What have you tried so far ?
    $endgroup$
    – Digitalis
    Dec 3 '18 at 21:24


















  • $begingroup$
    What have you tried so far ?
    $endgroup$
    – Digitalis
    Dec 3 '18 at 21:24
















$begingroup$
What have you tried so far ?
$endgroup$
– Digitalis
Dec 3 '18 at 21:24




$begingroup$
What have you tried so far ?
$endgroup$
– Digitalis
Dec 3 '18 at 21:24










4 Answers
4






active

oldest

votes


















3












$begingroup$

Hint: you can use the fact the sequence converges to say the inf of all the numbers past some point $N$ is greater than, say, half the limit. Just get $N$ from whoever told you the sequence converges by giving them $epsilon$ as half the limit. Now there are only finitely many numbers out to $N$.






share|cite|improve this answer









$endgroup$













  • $begingroup$
    Doesn't that assume it's a decreasing sequence?
    $endgroup$
    – timtfj
    Dec 3 '18 at 21:33










  • $begingroup$
    @timtfj: Not at all. As long as the sequence converges to $L$, there is some $N$ so that all the terms past $N$ are within $(frac L2, frac {3L}2)$ by using the $epsilon-N$ definition with $epsilon=frac L2$. It can bounce around however it wants.
    $endgroup$
    – Ross Millikan
    Dec 3 '18 at 21:36










  • $begingroup$
    Maybe I've misunderstood what you mean, but II if for example it converges upwards to the infimum of the sequence, the whole sequence is below its own infimum, surely? So the infimum isn't the infimum.
    $endgroup$
    – timtfj
    Dec 3 '18 at 21:41












  • $begingroup$
    @timtfj: No the inf of the sequence is the largest number that is less than or equal to all the terms of the sequence. If it is increasing all the terms are below the limit. If the sequence is $1-frac 1n$ for $n=3,4,5,ldots$ the limit is $1$ but the inf is $frac 23$
    $endgroup$
    – Ross Millikan
    Dec 3 '18 at 21:44










  • $begingroup$
    My problem is that the first sentence of your answer, seems to say that the infimum (past some N) is the limit. Am I misunderstanding?
    $endgroup$
    – timtfj
    Dec 3 '18 at 21:53





















1












$begingroup$

Outline of what I'd try:



The limit can't be negative, since then terms approaching it would be negative. Its not allowed to be zero, so is positive.



Call the sequence $(x_n)$ and the limit $k$. Choose an $epsilon$ such that $k-epsilon >0$.



Beyond some $n=N$, all $x_n>k-epsilon$ because of the convergence.



The first $N$ terms are all positive by definition, and have a minimum value $m$.



If the infimum is below both $m$ and $k-epsilon$ it can't meet the definition of an infimum. So it's $ge$ the lower of them, which is positive. Therefore the infimum is positive.



I think that does it, once it's translated into more precise language. (My aim here is to make it as human-readable as I can, to make clear how it works.)






share|cite|improve this answer











$endgroup$













  • $begingroup$
    This is an elaboration of my hint. I thought it better to leave more to OP.
    $endgroup$
    – Ross Millikan
    Dec 4 '18 at 3:34










  • $begingroup$
    It is, but as I totally misread your hint I didn't know I was elaborating it. I only realised the correct reading just now. My initial version of this was pretty vague and and in the end I decided enough hours had gone past for more detail to be OK.
    $endgroup$
    – timtfj
    Dec 4 '18 at 3:38





















0












$begingroup$

Let $(x_n)$ be a sequence of positive real numbers such that $ displaystyle lim_{n to infty} x_n = L neq 0 $. As $ frac{|L|}{2}>0 $, by definition, there exists $ N in mathbb{N} $ such that, for all $ n in mathbb{N} $, if $ n>N $, then $ |x_n - L|< frac{|L|}{2} $.



Let $ A = { 0,1,2,3,...,N } subset mathbb{N} $, $ B = mathbb{N} setminus A = { N+1, N+2, N+3, ... } subset mathbb{N} $ and $$m= min { x_0 , x_1, x_2 , x_3 , ... , x_N } .$$ Then $ mathbb{N} = A cup B $ and $ A cap B = varnothing $. Hence, for all $ n in mathbb{N} , $, we have either $ $ (i) $ n in A $ or $ $ (ii) $ n in B $, for which




(i) $ $ if $ n in A $, then $ 0 < m leq x_n $ ;




and




(ii) $ $ if $ n in B $, then $ n in mathbb{N} $ and $ n >N $ and we have that $ |x_n - L|< frac{|L|}{2} $. Therefore $$|L| -|x_n| leq big| |L| - |x_n| big| leq |L - x_n| = |x_n - L| < frac{|L|}{2} , $$ that is, $$x_n = |x_n|>|L|- frac{|L|}{2}= frac{|L|}{2} >0 . $$




Choosing $ c = min left{ m , frac{|L|}{2} right} $, we get that $ x_n geq c>0 $, $forall n in mathbb{N}$. So $ c>0 $ and $c$ is a lower bound for $(x_n)$. We conclude that $ displaystyle inf_{n in mathbb{N}} x_n geq c > 0 $.






share|cite|improve this answer









$endgroup$













  • $begingroup$
    This is what I hoped I could get OP to produce.
    $endgroup$
    – Ross Millikan
    Dec 4 '18 at 3:33



















0












$begingroup$

When studying new mathematical material, an attempt must be made be to write down 'the setup' in a mathematically precise way, and to avoid using logical arguments that use the informal language of, say, English.



The setup:



$tag 1 limlimits_{n to +∞} a_n = L, text{ with } L gt 0 text{ and } a_n gt 0$



$tag 2 A = { a_n , | , n in mathbb N}$



$tag 3 b = text{inf}(A)$



Assume $b = 0$.



Using open intervals of the form $(0, varepsilon)$, a strictly decreasing sequence $b_n$ with $b_n in A$ can be constructed such that



$tag 4 limlimits_{n to +∞} b_n = 0$



Now $b_n$ might not be a subsequence of $a_n$, but you can take another pass over it, throwing out terms if necessary to get the desired subsequence. Since every subsequence of $(a_n)_{ , n in mathbb N} $must have the same limit, $L = 0$, we arrive at a contradiction.



Note that you can construct $b_n$ as a subsequence on the first pass, but thought it might be easier to follow by breaking it into two steps. The basic truth is that for any finite set of positive numbers, an $varepsilon gt 0$ can be found such that the open interval $(0, varepsilon)$ is disjoint from that set.





The OP states in bold




it means the limit of the series is zero (and the sequence must be
decreasing)




Let $(a_n)_{ , n in mathbb N} $ be defined by



$$
a_n = left{begin{array}{lr}
frac{1}{n+1}, & text{for } n text{ even }\
frac{1}{n-.5}, & text{for } n text{ odd }
end{array}right}
$$



This sequence of positive numbers converges to $0$, but it is not a decreasing sequence

(see the wikipedia definition).






share|cite|improve this answer











$endgroup$













    Your Answer





    StackExchange.ifUsing("editor", function () {
    return StackExchange.using("mathjaxEditing", function () {
    StackExchange.MarkdownEditor.creationCallbacks.add(function (editor, postfix) {
    StackExchange.mathjaxEditing.prepareWmdForMathJax(editor, postfix, [["$", "$"], ["\\(","\\)"]]);
    });
    });
    }, "mathjax-editing");

    StackExchange.ready(function() {
    var channelOptions = {
    tags: "".split(" "),
    id: "69"
    };
    initTagRenderer("".split(" "), "".split(" "), channelOptions);

    StackExchange.using("externalEditor", function() {
    // Have to fire editor after snippets, if snippets enabled
    if (StackExchange.settings.snippets.snippetsEnabled) {
    StackExchange.using("snippets", function() {
    createEditor();
    });
    }
    else {
    createEditor();
    }
    });

    function createEditor() {
    StackExchange.prepareEditor({
    heartbeatType: 'answer',
    autoActivateHeartbeat: false,
    convertImagesToLinks: true,
    noModals: true,
    showLowRepImageUploadWarning: true,
    reputationToPostImages: 10,
    bindNavPrevention: true,
    postfix: "",
    imageUploader: {
    brandingHtml: "Powered by u003ca class="icon-imgur-white" href="https://imgur.com/"u003eu003c/au003e",
    contentPolicyHtml: "User contributions licensed under u003ca href="https://creativecommons.org/licenses/by-sa/3.0/"u003ecc by-sa 3.0 with attribution requiredu003c/au003e u003ca href="https://stackoverflow.com/legal/content-policy"u003e(content policy)u003c/au003e",
    allowUrls: true
    },
    noCode: true, onDemand: true,
    discardSelector: ".discard-answer"
    ,immediatelyShowMarkdownHelp:true
    });


    }
    });














    draft saved

    draft discarded


















    StackExchange.ready(
    function () {
    StackExchange.openid.initPostLogin('.new-post-login', 'https%3a%2f%2fmath.stackexchange.com%2fquestions%2f3024692%2ffor-a-sequence-of-positive-real-numbers-converging-to-a-limit-which-is-not-equa%23new-answer', 'question_page');
    }
    );

    Post as a guest















    Required, but never shown

























    4 Answers
    4






    active

    oldest

    votes








    4 Answers
    4






    active

    oldest

    votes









    active

    oldest

    votes






    active

    oldest

    votes









    3












    $begingroup$

    Hint: you can use the fact the sequence converges to say the inf of all the numbers past some point $N$ is greater than, say, half the limit. Just get $N$ from whoever told you the sequence converges by giving them $epsilon$ as half the limit. Now there are only finitely many numbers out to $N$.






    share|cite|improve this answer









    $endgroup$













    • $begingroup$
      Doesn't that assume it's a decreasing sequence?
      $endgroup$
      – timtfj
      Dec 3 '18 at 21:33










    • $begingroup$
      @timtfj: Not at all. As long as the sequence converges to $L$, there is some $N$ so that all the terms past $N$ are within $(frac L2, frac {3L}2)$ by using the $epsilon-N$ definition with $epsilon=frac L2$. It can bounce around however it wants.
      $endgroup$
      – Ross Millikan
      Dec 3 '18 at 21:36










    • $begingroup$
      Maybe I've misunderstood what you mean, but II if for example it converges upwards to the infimum of the sequence, the whole sequence is below its own infimum, surely? So the infimum isn't the infimum.
      $endgroup$
      – timtfj
      Dec 3 '18 at 21:41












    • $begingroup$
      @timtfj: No the inf of the sequence is the largest number that is less than or equal to all the terms of the sequence. If it is increasing all the terms are below the limit. If the sequence is $1-frac 1n$ for $n=3,4,5,ldots$ the limit is $1$ but the inf is $frac 23$
      $endgroup$
      – Ross Millikan
      Dec 3 '18 at 21:44










    • $begingroup$
      My problem is that the first sentence of your answer, seems to say that the infimum (past some N) is the limit. Am I misunderstanding?
      $endgroup$
      – timtfj
      Dec 3 '18 at 21:53


















    3












    $begingroup$

    Hint: you can use the fact the sequence converges to say the inf of all the numbers past some point $N$ is greater than, say, half the limit. Just get $N$ from whoever told you the sequence converges by giving them $epsilon$ as half the limit. Now there are only finitely many numbers out to $N$.






    share|cite|improve this answer









    $endgroup$













    • $begingroup$
      Doesn't that assume it's a decreasing sequence?
      $endgroup$
      – timtfj
      Dec 3 '18 at 21:33










    • $begingroup$
      @timtfj: Not at all. As long as the sequence converges to $L$, there is some $N$ so that all the terms past $N$ are within $(frac L2, frac {3L}2)$ by using the $epsilon-N$ definition with $epsilon=frac L2$. It can bounce around however it wants.
      $endgroup$
      – Ross Millikan
      Dec 3 '18 at 21:36










    • $begingroup$
      Maybe I've misunderstood what you mean, but II if for example it converges upwards to the infimum of the sequence, the whole sequence is below its own infimum, surely? So the infimum isn't the infimum.
      $endgroup$
      – timtfj
      Dec 3 '18 at 21:41












    • $begingroup$
      @timtfj: No the inf of the sequence is the largest number that is less than or equal to all the terms of the sequence. If it is increasing all the terms are below the limit. If the sequence is $1-frac 1n$ for $n=3,4,5,ldots$ the limit is $1$ but the inf is $frac 23$
      $endgroup$
      – Ross Millikan
      Dec 3 '18 at 21:44










    • $begingroup$
      My problem is that the first sentence of your answer, seems to say that the infimum (past some N) is the limit. Am I misunderstanding?
      $endgroup$
      – timtfj
      Dec 3 '18 at 21:53
















    3












    3








    3





    $begingroup$

    Hint: you can use the fact the sequence converges to say the inf of all the numbers past some point $N$ is greater than, say, half the limit. Just get $N$ from whoever told you the sequence converges by giving them $epsilon$ as half the limit. Now there are only finitely many numbers out to $N$.






    share|cite|improve this answer









    $endgroup$



    Hint: you can use the fact the sequence converges to say the inf of all the numbers past some point $N$ is greater than, say, half the limit. Just get $N$ from whoever told you the sequence converges by giving them $epsilon$ as half the limit. Now there are only finitely many numbers out to $N$.







    share|cite|improve this answer












    share|cite|improve this answer



    share|cite|improve this answer










    answered Dec 3 '18 at 21:29









    Ross MillikanRoss Millikan

    297k23198371




    297k23198371












    • $begingroup$
      Doesn't that assume it's a decreasing sequence?
      $endgroup$
      – timtfj
      Dec 3 '18 at 21:33










    • $begingroup$
      @timtfj: Not at all. As long as the sequence converges to $L$, there is some $N$ so that all the terms past $N$ are within $(frac L2, frac {3L}2)$ by using the $epsilon-N$ definition with $epsilon=frac L2$. It can bounce around however it wants.
      $endgroup$
      – Ross Millikan
      Dec 3 '18 at 21:36










    • $begingroup$
      Maybe I've misunderstood what you mean, but II if for example it converges upwards to the infimum of the sequence, the whole sequence is below its own infimum, surely? So the infimum isn't the infimum.
      $endgroup$
      – timtfj
      Dec 3 '18 at 21:41












    • $begingroup$
      @timtfj: No the inf of the sequence is the largest number that is less than or equal to all the terms of the sequence. If it is increasing all the terms are below the limit. If the sequence is $1-frac 1n$ for $n=3,4,5,ldots$ the limit is $1$ but the inf is $frac 23$
      $endgroup$
      – Ross Millikan
      Dec 3 '18 at 21:44










    • $begingroup$
      My problem is that the first sentence of your answer, seems to say that the infimum (past some N) is the limit. Am I misunderstanding?
      $endgroup$
      – timtfj
      Dec 3 '18 at 21:53




















    • $begingroup$
      Doesn't that assume it's a decreasing sequence?
      $endgroup$
      – timtfj
      Dec 3 '18 at 21:33










    • $begingroup$
      @timtfj: Not at all. As long as the sequence converges to $L$, there is some $N$ so that all the terms past $N$ are within $(frac L2, frac {3L}2)$ by using the $epsilon-N$ definition with $epsilon=frac L2$. It can bounce around however it wants.
      $endgroup$
      – Ross Millikan
      Dec 3 '18 at 21:36










    • $begingroup$
      Maybe I've misunderstood what you mean, but II if for example it converges upwards to the infimum of the sequence, the whole sequence is below its own infimum, surely? So the infimum isn't the infimum.
      $endgroup$
      – timtfj
      Dec 3 '18 at 21:41












    • $begingroup$
      @timtfj: No the inf of the sequence is the largest number that is less than or equal to all the terms of the sequence. If it is increasing all the terms are below the limit. If the sequence is $1-frac 1n$ for $n=3,4,5,ldots$ the limit is $1$ but the inf is $frac 23$
      $endgroup$
      – Ross Millikan
      Dec 3 '18 at 21:44










    • $begingroup$
      My problem is that the first sentence of your answer, seems to say that the infimum (past some N) is the limit. Am I misunderstanding?
      $endgroup$
      – timtfj
      Dec 3 '18 at 21:53


















    $begingroup$
    Doesn't that assume it's a decreasing sequence?
    $endgroup$
    – timtfj
    Dec 3 '18 at 21:33




    $begingroup$
    Doesn't that assume it's a decreasing sequence?
    $endgroup$
    – timtfj
    Dec 3 '18 at 21:33












    $begingroup$
    @timtfj: Not at all. As long as the sequence converges to $L$, there is some $N$ so that all the terms past $N$ are within $(frac L2, frac {3L}2)$ by using the $epsilon-N$ definition with $epsilon=frac L2$. It can bounce around however it wants.
    $endgroup$
    – Ross Millikan
    Dec 3 '18 at 21:36




    $begingroup$
    @timtfj: Not at all. As long as the sequence converges to $L$, there is some $N$ so that all the terms past $N$ are within $(frac L2, frac {3L}2)$ by using the $epsilon-N$ definition with $epsilon=frac L2$. It can bounce around however it wants.
    $endgroup$
    – Ross Millikan
    Dec 3 '18 at 21:36












    $begingroup$
    Maybe I've misunderstood what you mean, but II if for example it converges upwards to the infimum of the sequence, the whole sequence is below its own infimum, surely? So the infimum isn't the infimum.
    $endgroup$
    – timtfj
    Dec 3 '18 at 21:41






    $begingroup$
    Maybe I've misunderstood what you mean, but II if for example it converges upwards to the infimum of the sequence, the whole sequence is below its own infimum, surely? So the infimum isn't the infimum.
    $endgroup$
    – timtfj
    Dec 3 '18 at 21:41














    $begingroup$
    @timtfj: No the inf of the sequence is the largest number that is less than or equal to all the terms of the sequence. If it is increasing all the terms are below the limit. If the sequence is $1-frac 1n$ for $n=3,4,5,ldots$ the limit is $1$ but the inf is $frac 23$
    $endgroup$
    – Ross Millikan
    Dec 3 '18 at 21:44




    $begingroup$
    @timtfj: No the inf of the sequence is the largest number that is less than or equal to all the terms of the sequence. If it is increasing all the terms are below the limit. If the sequence is $1-frac 1n$ for $n=3,4,5,ldots$ the limit is $1$ but the inf is $frac 23$
    $endgroup$
    – Ross Millikan
    Dec 3 '18 at 21:44












    $begingroup$
    My problem is that the first sentence of your answer, seems to say that the infimum (past some N) is the limit. Am I misunderstanding?
    $endgroup$
    – timtfj
    Dec 3 '18 at 21:53






    $begingroup$
    My problem is that the first sentence of your answer, seems to say that the infimum (past some N) is the limit. Am I misunderstanding?
    $endgroup$
    – timtfj
    Dec 3 '18 at 21:53













    1












    $begingroup$

    Outline of what I'd try:



    The limit can't be negative, since then terms approaching it would be negative. Its not allowed to be zero, so is positive.



    Call the sequence $(x_n)$ and the limit $k$. Choose an $epsilon$ such that $k-epsilon >0$.



    Beyond some $n=N$, all $x_n>k-epsilon$ because of the convergence.



    The first $N$ terms are all positive by definition, and have a minimum value $m$.



    If the infimum is below both $m$ and $k-epsilon$ it can't meet the definition of an infimum. So it's $ge$ the lower of them, which is positive. Therefore the infimum is positive.



    I think that does it, once it's translated into more precise language. (My aim here is to make it as human-readable as I can, to make clear how it works.)






    share|cite|improve this answer











    $endgroup$













    • $begingroup$
      This is an elaboration of my hint. I thought it better to leave more to OP.
      $endgroup$
      – Ross Millikan
      Dec 4 '18 at 3:34










    • $begingroup$
      It is, but as I totally misread your hint I didn't know I was elaborating it. I only realised the correct reading just now. My initial version of this was pretty vague and and in the end I decided enough hours had gone past for more detail to be OK.
      $endgroup$
      – timtfj
      Dec 4 '18 at 3:38


















    1












    $begingroup$

    Outline of what I'd try:



    The limit can't be negative, since then terms approaching it would be negative. Its not allowed to be zero, so is positive.



    Call the sequence $(x_n)$ and the limit $k$. Choose an $epsilon$ such that $k-epsilon >0$.



    Beyond some $n=N$, all $x_n>k-epsilon$ because of the convergence.



    The first $N$ terms are all positive by definition, and have a minimum value $m$.



    If the infimum is below both $m$ and $k-epsilon$ it can't meet the definition of an infimum. So it's $ge$ the lower of them, which is positive. Therefore the infimum is positive.



    I think that does it, once it's translated into more precise language. (My aim here is to make it as human-readable as I can, to make clear how it works.)






    share|cite|improve this answer











    $endgroup$













    • $begingroup$
      This is an elaboration of my hint. I thought it better to leave more to OP.
      $endgroup$
      – Ross Millikan
      Dec 4 '18 at 3:34










    • $begingroup$
      It is, but as I totally misread your hint I didn't know I was elaborating it. I only realised the correct reading just now. My initial version of this was pretty vague and and in the end I decided enough hours had gone past for more detail to be OK.
      $endgroup$
      – timtfj
      Dec 4 '18 at 3:38
















    1












    1








    1





    $begingroup$

    Outline of what I'd try:



    The limit can't be negative, since then terms approaching it would be negative. Its not allowed to be zero, so is positive.



    Call the sequence $(x_n)$ and the limit $k$. Choose an $epsilon$ such that $k-epsilon >0$.



    Beyond some $n=N$, all $x_n>k-epsilon$ because of the convergence.



    The first $N$ terms are all positive by definition, and have a minimum value $m$.



    If the infimum is below both $m$ and $k-epsilon$ it can't meet the definition of an infimum. So it's $ge$ the lower of them, which is positive. Therefore the infimum is positive.



    I think that does it, once it's translated into more precise language. (My aim here is to make it as human-readable as I can, to make clear how it works.)






    share|cite|improve this answer











    $endgroup$



    Outline of what I'd try:



    The limit can't be negative, since then terms approaching it would be negative. Its not allowed to be zero, so is positive.



    Call the sequence $(x_n)$ and the limit $k$. Choose an $epsilon$ such that $k-epsilon >0$.



    Beyond some $n=N$, all $x_n>k-epsilon$ because of the convergence.



    The first $N$ terms are all positive by definition, and have a minimum value $m$.



    If the infimum is below both $m$ and $k-epsilon$ it can't meet the definition of an infimum. So it's $ge$ the lower of them, which is positive. Therefore the infimum is positive.



    I think that does it, once it's translated into more precise language. (My aim here is to make it as human-readable as I can, to make clear how it works.)







    share|cite|improve this answer














    share|cite|improve this answer



    share|cite|improve this answer








    edited Dec 4 '18 at 3:32

























    answered Dec 3 '18 at 22:10









    timtfjtimtfj

    2,448420




    2,448420












    • $begingroup$
      This is an elaboration of my hint. I thought it better to leave more to OP.
      $endgroup$
      – Ross Millikan
      Dec 4 '18 at 3:34










    • $begingroup$
      It is, but as I totally misread your hint I didn't know I was elaborating it. I only realised the correct reading just now. My initial version of this was pretty vague and and in the end I decided enough hours had gone past for more detail to be OK.
      $endgroup$
      – timtfj
      Dec 4 '18 at 3:38




















    • $begingroup$
      This is an elaboration of my hint. I thought it better to leave more to OP.
      $endgroup$
      – Ross Millikan
      Dec 4 '18 at 3:34










    • $begingroup$
      It is, but as I totally misread your hint I didn't know I was elaborating it. I only realised the correct reading just now. My initial version of this was pretty vague and and in the end I decided enough hours had gone past for more detail to be OK.
      $endgroup$
      – timtfj
      Dec 4 '18 at 3:38


















    $begingroup$
    This is an elaboration of my hint. I thought it better to leave more to OP.
    $endgroup$
    – Ross Millikan
    Dec 4 '18 at 3:34




    $begingroup$
    This is an elaboration of my hint. I thought it better to leave more to OP.
    $endgroup$
    – Ross Millikan
    Dec 4 '18 at 3:34












    $begingroup$
    It is, but as I totally misread your hint I didn't know I was elaborating it. I only realised the correct reading just now. My initial version of this was pretty vague and and in the end I decided enough hours had gone past for more detail to be OK.
    $endgroup$
    – timtfj
    Dec 4 '18 at 3:38






    $begingroup$
    It is, but as I totally misread your hint I didn't know I was elaborating it. I only realised the correct reading just now. My initial version of this was pretty vague and and in the end I decided enough hours had gone past for more detail to be OK.
    $endgroup$
    – timtfj
    Dec 4 '18 at 3:38













    0












    $begingroup$

    Let $(x_n)$ be a sequence of positive real numbers such that $ displaystyle lim_{n to infty} x_n = L neq 0 $. As $ frac{|L|}{2}>0 $, by definition, there exists $ N in mathbb{N} $ such that, for all $ n in mathbb{N} $, if $ n>N $, then $ |x_n - L|< frac{|L|}{2} $.



    Let $ A = { 0,1,2,3,...,N } subset mathbb{N} $, $ B = mathbb{N} setminus A = { N+1, N+2, N+3, ... } subset mathbb{N} $ and $$m= min { x_0 , x_1, x_2 , x_3 , ... , x_N } .$$ Then $ mathbb{N} = A cup B $ and $ A cap B = varnothing $. Hence, for all $ n in mathbb{N} , $, we have either $ $ (i) $ n in A $ or $ $ (ii) $ n in B $, for which




    (i) $ $ if $ n in A $, then $ 0 < m leq x_n $ ;




    and




    (ii) $ $ if $ n in B $, then $ n in mathbb{N} $ and $ n >N $ and we have that $ |x_n - L|< frac{|L|}{2} $. Therefore $$|L| -|x_n| leq big| |L| - |x_n| big| leq |L - x_n| = |x_n - L| < frac{|L|}{2} , $$ that is, $$x_n = |x_n|>|L|- frac{|L|}{2}= frac{|L|}{2} >0 . $$




    Choosing $ c = min left{ m , frac{|L|}{2} right} $, we get that $ x_n geq c>0 $, $forall n in mathbb{N}$. So $ c>0 $ and $c$ is a lower bound for $(x_n)$. We conclude that $ displaystyle inf_{n in mathbb{N}} x_n geq c > 0 $.






    share|cite|improve this answer









    $endgroup$













    • $begingroup$
      This is what I hoped I could get OP to produce.
      $endgroup$
      – Ross Millikan
      Dec 4 '18 at 3:33
















    0












    $begingroup$

    Let $(x_n)$ be a sequence of positive real numbers such that $ displaystyle lim_{n to infty} x_n = L neq 0 $. As $ frac{|L|}{2}>0 $, by definition, there exists $ N in mathbb{N} $ such that, for all $ n in mathbb{N} $, if $ n>N $, then $ |x_n - L|< frac{|L|}{2} $.



    Let $ A = { 0,1,2,3,...,N } subset mathbb{N} $, $ B = mathbb{N} setminus A = { N+1, N+2, N+3, ... } subset mathbb{N} $ and $$m= min { x_0 , x_1, x_2 , x_3 , ... , x_N } .$$ Then $ mathbb{N} = A cup B $ and $ A cap B = varnothing $. Hence, for all $ n in mathbb{N} , $, we have either $ $ (i) $ n in A $ or $ $ (ii) $ n in B $, for which




    (i) $ $ if $ n in A $, then $ 0 < m leq x_n $ ;




    and




    (ii) $ $ if $ n in B $, then $ n in mathbb{N} $ and $ n >N $ and we have that $ |x_n - L|< frac{|L|}{2} $. Therefore $$|L| -|x_n| leq big| |L| - |x_n| big| leq |L - x_n| = |x_n - L| < frac{|L|}{2} , $$ that is, $$x_n = |x_n|>|L|- frac{|L|}{2}= frac{|L|}{2} >0 . $$




    Choosing $ c = min left{ m , frac{|L|}{2} right} $, we get that $ x_n geq c>0 $, $forall n in mathbb{N}$. So $ c>0 $ and $c$ is a lower bound for $(x_n)$. We conclude that $ displaystyle inf_{n in mathbb{N}} x_n geq c > 0 $.






    share|cite|improve this answer









    $endgroup$













    • $begingroup$
      This is what I hoped I could get OP to produce.
      $endgroup$
      – Ross Millikan
      Dec 4 '18 at 3:33














    0












    0








    0





    $begingroup$

    Let $(x_n)$ be a sequence of positive real numbers such that $ displaystyle lim_{n to infty} x_n = L neq 0 $. As $ frac{|L|}{2}>0 $, by definition, there exists $ N in mathbb{N} $ such that, for all $ n in mathbb{N} $, if $ n>N $, then $ |x_n - L|< frac{|L|}{2} $.



    Let $ A = { 0,1,2,3,...,N } subset mathbb{N} $, $ B = mathbb{N} setminus A = { N+1, N+2, N+3, ... } subset mathbb{N} $ and $$m= min { x_0 , x_1, x_2 , x_3 , ... , x_N } .$$ Then $ mathbb{N} = A cup B $ and $ A cap B = varnothing $. Hence, for all $ n in mathbb{N} , $, we have either $ $ (i) $ n in A $ or $ $ (ii) $ n in B $, for which




    (i) $ $ if $ n in A $, then $ 0 < m leq x_n $ ;




    and




    (ii) $ $ if $ n in B $, then $ n in mathbb{N} $ and $ n >N $ and we have that $ |x_n - L|< frac{|L|}{2} $. Therefore $$|L| -|x_n| leq big| |L| - |x_n| big| leq |L - x_n| = |x_n - L| < frac{|L|}{2} , $$ that is, $$x_n = |x_n|>|L|- frac{|L|}{2}= frac{|L|}{2} >0 . $$




    Choosing $ c = min left{ m , frac{|L|}{2} right} $, we get that $ x_n geq c>0 $, $forall n in mathbb{N}$. So $ c>0 $ and $c$ is a lower bound for $(x_n)$. We conclude that $ displaystyle inf_{n in mathbb{N}} x_n geq c > 0 $.






    share|cite|improve this answer









    $endgroup$



    Let $(x_n)$ be a sequence of positive real numbers such that $ displaystyle lim_{n to infty} x_n = L neq 0 $. As $ frac{|L|}{2}>0 $, by definition, there exists $ N in mathbb{N} $ such that, for all $ n in mathbb{N} $, if $ n>N $, then $ |x_n - L|< frac{|L|}{2} $.



    Let $ A = { 0,1,2,3,...,N } subset mathbb{N} $, $ B = mathbb{N} setminus A = { N+1, N+2, N+3, ... } subset mathbb{N} $ and $$m= min { x_0 , x_1, x_2 , x_3 , ... , x_N } .$$ Then $ mathbb{N} = A cup B $ and $ A cap B = varnothing $. Hence, for all $ n in mathbb{N} , $, we have either $ $ (i) $ n in A $ or $ $ (ii) $ n in B $, for which




    (i) $ $ if $ n in A $, then $ 0 < m leq x_n $ ;




    and




    (ii) $ $ if $ n in B $, then $ n in mathbb{N} $ and $ n >N $ and we have that $ |x_n - L|< frac{|L|}{2} $. Therefore $$|L| -|x_n| leq big| |L| - |x_n| big| leq |L - x_n| = |x_n - L| < frac{|L|}{2} , $$ that is, $$x_n = |x_n|>|L|- frac{|L|}{2}= frac{|L|}{2} >0 . $$




    Choosing $ c = min left{ m , frac{|L|}{2} right} $, we get that $ x_n geq c>0 $, $forall n in mathbb{N}$. So $ c>0 $ and $c$ is a lower bound for $(x_n)$. We conclude that $ displaystyle inf_{n in mathbb{N}} x_n geq c > 0 $.







    share|cite|improve this answer












    share|cite|improve this answer



    share|cite|improve this answer










    answered Dec 3 '18 at 22:15









    GustavoGustavo

    866621




    866621












    • $begingroup$
      This is what I hoped I could get OP to produce.
      $endgroup$
      – Ross Millikan
      Dec 4 '18 at 3:33


















    • $begingroup$
      This is what I hoped I could get OP to produce.
      $endgroup$
      – Ross Millikan
      Dec 4 '18 at 3:33
















    $begingroup$
    This is what I hoped I could get OP to produce.
    $endgroup$
    – Ross Millikan
    Dec 4 '18 at 3:33




    $begingroup$
    This is what I hoped I could get OP to produce.
    $endgroup$
    – Ross Millikan
    Dec 4 '18 at 3:33











    0












    $begingroup$

    When studying new mathematical material, an attempt must be made be to write down 'the setup' in a mathematically precise way, and to avoid using logical arguments that use the informal language of, say, English.



    The setup:



    $tag 1 limlimits_{n to +∞} a_n = L, text{ with } L gt 0 text{ and } a_n gt 0$



    $tag 2 A = { a_n , | , n in mathbb N}$



    $tag 3 b = text{inf}(A)$



    Assume $b = 0$.



    Using open intervals of the form $(0, varepsilon)$, a strictly decreasing sequence $b_n$ with $b_n in A$ can be constructed such that



    $tag 4 limlimits_{n to +∞} b_n = 0$



    Now $b_n$ might not be a subsequence of $a_n$, but you can take another pass over it, throwing out terms if necessary to get the desired subsequence. Since every subsequence of $(a_n)_{ , n in mathbb N} $must have the same limit, $L = 0$, we arrive at a contradiction.



    Note that you can construct $b_n$ as a subsequence on the first pass, but thought it might be easier to follow by breaking it into two steps. The basic truth is that for any finite set of positive numbers, an $varepsilon gt 0$ can be found such that the open interval $(0, varepsilon)$ is disjoint from that set.





    The OP states in bold




    it means the limit of the series is zero (and the sequence must be
    decreasing)




    Let $(a_n)_{ , n in mathbb N} $ be defined by



    $$
    a_n = left{begin{array}{lr}
    frac{1}{n+1}, & text{for } n text{ even }\
    frac{1}{n-.5}, & text{for } n text{ odd }
    end{array}right}
    $$



    This sequence of positive numbers converges to $0$, but it is not a decreasing sequence

    (see the wikipedia definition).






    share|cite|improve this answer











    $endgroup$


















      0












      $begingroup$

      When studying new mathematical material, an attempt must be made be to write down 'the setup' in a mathematically precise way, and to avoid using logical arguments that use the informal language of, say, English.



      The setup:



      $tag 1 limlimits_{n to +∞} a_n = L, text{ with } L gt 0 text{ and } a_n gt 0$



      $tag 2 A = { a_n , | , n in mathbb N}$



      $tag 3 b = text{inf}(A)$



      Assume $b = 0$.



      Using open intervals of the form $(0, varepsilon)$, a strictly decreasing sequence $b_n$ with $b_n in A$ can be constructed such that



      $tag 4 limlimits_{n to +∞} b_n = 0$



      Now $b_n$ might not be a subsequence of $a_n$, but you can take another pass over it, throwing out terms if necessary to get the desired subsequence. Since every subsequence of $(a_n)_{ , n in mathbb N} $must have the same limit, $L = 0$, we arrive at a contradiction.



      Note that you can construct $b_n$ as a subsequence on the first pass, but thought it might be easier to follow by breaking it into two steps. The basic truth is that for any finite set of positive numbers, an $varepsilon gt 0$ can be found such that the open interval $(0, varepsilon)$ is disjoint from that set.





      The OP states in bold




      it means the limit of the series is zero (and the sequence must be
      decreasing)




      Let $(a_n)_{ , n in mathbb N} $ be defined by



      $$
      a_n = left{begin{array}{lr}
      frac{1}{n+1}, & text{for } n text{ even }\
      frac{1}{n-.5}, & text{for } n text{ odd }
      end{array}right}
      $$



      This sequence of positive numbers converges to $0$, but it is not a decreasing sequence

      (see the wikipedia definition).






      share|cite|improve this answer











      $endgroup$
















        0












        0








        0





        $begingroup$

        When studying new mathematical material, an attempt must be made be to write down 'the setup' in a mathematically precise way, and to avoid using logical arguments that use the informal language of, say, English.



        The setup:



        $tag 1 limlimits_{n to +∞} a_n = L, text{ with } L gt 0 text{ and } a_n gt 0$



        $tag 2 A = { a_n , | , n in mathbb N}$



        $tag 3 b = text{inf}(A)$



        Assume $b = 0$.



        Using open intervals of the form $(0, varepsilon)$, a strictly decreasing sequence $b_n$ with $b_n in A$ can be constructed such that



        $tag 4 limlimits_{n to +∞} b_n = 0$



        Now $b_n$ might not be a subsequence of $a_n$, but you can take another pass over it, throwing out terms if necessary to get the desired subsequence. Since every subsequence of $(a_n)_{ , n in mathbb N} $must have the same limit, $L = 0$, we arrive at a contradiction.



        Note that you can construct $b_n$ as a subsequence on the first pass, but thought it might be easier to follow by breaking it into two steps. The basic truth is that for any finite set of positive numbers, an $varepsilon gt 0$ can be found such that the open interval $(0, varepsilon)$ is disjoint from that set.





        The OP states in bold




        it means the limit of the series is zero (and the sequence must be
        decreasing)




        Let $(a_n)_{ , n in mathbb N} $ be defined by



        $$
        a_n = left{begin{array}{lr}
        frac{1}{n+1}, & text{for } n text{ even }\
        frac{1}{n-.5}, & text{for } n text{ odd }
        end{array}right}
        $$



        This sequence of positive numbers converges to $0$, but it is not a decreasing sequence

        (see the wikipedia definition).






        share|cite|improve this answer











        $endgroup$



        When studying new mathematical material, an attempt must be made be to write down 'the setup' in a mathematically precise way, and to avoid using logical arguments that use the informal language of, say, English.



        The setup:



        $tag 1 limlimits_{n to +∞} a_n = L, text{ with } L gt 0 text{ and } a_n gt 0$



        $tag 2 A = { a_n , | , n in mathbb N}$



        $tag 3 b = text{inf}(A)$



        Assume $b = 0$.



        Using open intervals of the form $(0, varepsilon)$, a strictly decreasing sequence $b_n$ with $b_n in A$ can be constructed such that



        $tag 4 limlimits_{n to +∞} b_n = 0$



        Now $b_n$ might not be a subsequence of $a_n$, but you can take another pass over it, throwing out terms if necessary to get the desired subsequence. Since every subsequence of $(a_n)_{ , n in mathbb N} $must have the same limit, $L = 0$, we arrive at a contradiction.



        Note that you can construct $b_n$ as a subsequence on the first pass, but thought it might be easier to follow by breaking it into two steps. The basic truth is that for any finite set of positive numbers, an $varepsilon gt 0$ can be found such that the open interval $(0, varepsilon)$ is disjoint from that set.





        The OP states in bold




        it means the limit of the series is zero (and the sequence must be
        decreasing)




        Let $(a_n)_{ , n in mathbb N} $ be defined by



        $$
        a_n = left{begin{array}{lr}
        frac{1}{n+1}, & text{for } n text{ even }\
        frac{1}{n-.5}, & text{for } n text{ odd }
        end{array}right}
        $$



        This sequence of positive numbers converges to $0$, but it is not a decreasing sequence

        (see the wikipedia definition).







        share|cite|improve this answer














        share|cite|improve this answer



        share|cite|improve this answer








        edited Dec 4 '18 at 17:48

























        answered Dec 4 '18 at 15:32









        CopyPasteItCopyPasteIt

        4,1781628




        4,1781628






























            draft saved

            draft discarded




















































            Thanks for contributing an answer to Mathematics Stack Exchange!


            • Please be sure to answer the question. Provide details and share your research!

            But avoid



            • Asking for help, clarification, or responding to other answers.

            • Making statements based on opinion; back them up with references or personal experience.


            Use MathJax to format equations. MathJax reference.


            To learn more, see our tips on writing great answers.




            draft saved


            draft discarded














            StackExchange.ready(
            function () {
            StackExchange.openid.initPostLogin('.new-post-login', 'https%3a%2f%2fmath.stackexchange.com%2fquestions%2f3024692%2ffor-a-sequence-of-positive-real-numbers-converging-to-a-limit-which-is-not-equa%23new-answer', 'question_page');
            }
            );

            Post as a guest















            Required, but never shown





















































            Required, but never shown














            Required, but never shown












            Required, but never shown







            Required, but never shown

































            Required, but never shown














            Required, but never shown












            Required, but never shown







            Required, but never shown







            Popular posts from this blog

            Biblatex bibliography style without URLs when DOI exists (in Overleaf with Zotero bibliography)

            ComboBox Display Member on multiple fields

            Is it possible to collect Nectar points via Trainline?